LSAT and Law School Admissions Forum

Get expert LSAT preparation and law school admissions advice from PowerScore Test Preparation.

 Administrator
PowerScore Staff
  • PowerScore Staff
  • Posts: 8917
  • Joined: Feb 02, 2011
|
#101006
Complete Question Explanation

Flaw in the Reasoning. The correct answer choice is (B).

Answer choice (A):

Answer choice (B): This is the correct answer choice.

Answer choice (C):

Answer choice (D):

Answer choice (E):
 jlam061695
  • Posts: 62
  • Joined: Sep 17, 2016
|
#30624
I initially picked D for this question, but I felt like both B and D could be correct? Is D incorrect because even though the evidence that the stimulus provided for the conclusion that "the party was not overambitious" was irrelevant, it still counts as evidence provided? If D had instead stated that the argument "fails to establish" that the revolutionary party's goals were not overambitious, would D have also been correct?
User avatar
 Jonathan Evans
PowerScore Staff
  • PowerScore Staff
  • Posts: 726
  • Joined: Jun 09, 2016
|
#30791
JLam,

You're on the right track. Answer choice (D) is incorrect because the Historian does in fact provide some evidence that the revolutionary party's goals were not overambitious ("most of the party's goals were quickly achieved"). There is still an error in the use of evidence flaw here. Some evidence that the goals were not overambitious does not suffice to establish that the party was not overambitious.

In contrast, answer choice (B) clearly describes the other major flaw here, that just because the party did not have enough power to cause the suffering it's critics claim it caused does not mean it caused no suffering.

Good question!
User avatar
 goingslow
  • Posts: 52
  • Joined: Aug 24, 2021
|
#97308
Hi there! I'd be very grateful if you could shed light on why (C) is incorrect.

Thank you!!
 Luke Haqq
PowerScore Staff
  • PowerScore Staff
  • Posts: 742
  • Joined: Apr 26, 2012
|
#97336
Hi goingslow!

Happy to address why answer choice (C) is incorrect.

We can start with the historian's conclusion: "So it is clear that the party was not overambitious and caused no suffering." How is this conclusion supported? Can you pre-phrase any flaws in how the author reaches it? One thing that was unclear in my reading was the author's move from the claim that a party didn't cause "great suffering" to the conclusion that it therefore "caused no suffering" whatsoever. This is a flaw, they could still have caused some suffering even if it didn't amount to great suffering. Something like this pre-phrase is captured in answer choice (B): the reasoning "fails to establish that the revolutionary party caused no suffering."

Answer choice (C) states that the reasoning "fails to establish that any of the revolutionary party's critics underestimated the party's power." This seems contrary to the information in the stimulus, for the author is suggesting that critics did underestimate the party's power in claiming that the party's goals were "overambitious" when it ended up achieving these goals quickly. In addition, we're told that the "revolutionary party has been accused of having many overambitious goals." In other words, these are the critics making the accusation--which seems to be establishing that some of them underestimate the party's power, given that it achieved these goals quickly.

Get the most out of your LSAT Prep Plus subscription.

Analyze and track your performance with our Testing and Analytics Package.